Középiskolai Matematikai és Fizikai Lapok
Informatika rovattal
Kiadja a MATFUND Alapítvány
Már regisztráltál?
Új vendég vagy?

Fórum: Érdekes matekfeladatok

  [1]    [2]    [3]    [4]    [5]    [6]    [7]    [8]    [9]    [10]    [11]    [12]    [13]    [14]    [15]    [16]    [17]    [18]    [19]    [20]    [21]    [22]    [23]    [24]    [25]    [26]    [27]    [28]    [29]    [30]    [31]    [32]    [33]    [34]    [35]    [36]    [37]    [38]    [39]    [40]    [41]    [42]    [43]    [44]    [45]    [46]    [47]    [48]    [49]    [50]    [51]    [52]    [53]    [54]    [55]    [56]    [57]    [58]    [59]    [60]    [61]    [62]    [63]    [64]    [65]    [66]    [67]    [68]    [69]    [70]    [71]    [72]    [73]    [74]    [75]    [76]    [77]    [78]    [79]    [80]    [81]    [82]    [83]    [84]    [85]    [86]    [87]    [88]    [89]    [90]    [91]    [92]    [93]    [94]    [95]    [96]    [97]    [98]    [99]    [100]    [101]    [102]    [103]    [104]    [105]    [106]    [107]    [108]    [109]    [110]    [111]    [112]    [113]    [114]    [115]    [116]    [117]    [118]    [119]    [120]    [121]    [122]    [123]    [124]    [125]    [126]    [127]    [128]    [129]    [130]    [131]    [132]    [133]    [134]    [135]    [136]    [137]    [138]    [139]    [140]    [141]    [142]    [143]    [144]    [145]    [146]    [147]    [148]    [149]    [150]    [151]    [152]    [153]    [154]    [155]    [156]    [157]    [158]    [159]    [160]    [161]  

Szeretnél hozzászólni? Jelentkezz be.
[1957] epsilon2007-04-01 12:10:27

(x,y)->x+i×y miért nem lenne izomorfizmus?

Előzmény: [1948] Lóczi Lajos, 2007-03-24 21:55:34
[1956] Cckek2007-03-31 20:27:37

Bizonyítsuk be, hogy ha egy gyűrűben az 1-(xy)2 elem invertálható, akkor 1-(yx)2 is invertálható.

[1954] Lóczi Lajos2007-03-30 00:01:09

és már csak össze kell számolni, hány darabból áll

Előzmény: [1953] Lóczi Lajos, 2007-03-29 23:56:31
[1953] Lóczi Lajos2007-03-29 23:56:31

90548514656103281165404177077484163874504589675413336841320

Előzmény: [1955] Cckek, 2007-03-29 20:11:02
[1955] Cckek2007-03-29 20:11:02

Hány számjegye van \binom{200}{100}-nak?

[1952] Cckek2007-03-27 20:26:03

Köszi.

Előzmény: [1950] Lóczi Lajos, 2007-03-27 10:42:56
[1951] BohnerGéza2007-03-27 14:49:56
Előzmény: [1946] Cckek, 2007-03-24 19:20:01
[1950] Lóczi Lajos2007-03-27 10:42:56

egy válasz

Előzmény: [1949] Cckek, 2007-03-27 10:19:18
[1949] Cckek2007-03-27 10:19:18

Bizonyítsuk be, hogy az (R,+) és (C,+) csoportok izomorfak.

[1948] Lóczi Lajos2007-03-24 21:55:34

Deriváljuk az integrált az x paraméter szerint, majd használjuk a jól ismert \cos(t)=\frac{1-u^2}{1+u^2} helyettesítést, ahol u=tan (t/2). Ekkor az u változó szerint egy racionális törtfüggvényt kapunk, aminek van elemi primitív függvénye. Visszahelyettesítve a t változót az alábbit kapjuk:

\frac{t + 2{\rm{arctan}} (\frac{\left( 1 + x \right) \tan (\frac{t}{2})}{-1 + x})}{x}.

Ennek a megváltozása kell t=0 és t=\pi között. A függvény t=0-ban 0, a t\to\pi- határesetben pedig x-től függően kétféle értéket vesz fel: vagy nullát vagy 2\pi/x-et. Már csak x szerint kell a primitív függvényt visszakeresni, ami persze triviális. (Az x=0, x=1, x=-1 eseteket persze külön meg kell vizsgálni az eredeti határozott integrálban.)

Végeredmény: az eredeti integrál értéke 0, ha -1\lex\le1, míg 2\pilog |x|, ha x>1 vagy x<-1.

Előzmény: [1947] Cckek, 2007-03-24 19:44:40
[1947] Cckek2007-03-24 19:44:40

312.feladat Ha már integráloknál tartunk:

Számítsuk ki a következő integrált:

\int_{0}^{\pi}\ln(1-2xcost+x^2)dt

[1946] Cckek2007-03-24 19:20:01

Legyen I=\int_{0}^{\frac{\pi}{4}}\sqrt{\tan x}dx

Jelöljük \sqrt{\tan x}=t, tehát

I=\int_{0}^{1}\frac{2t^2}{1+t^4}dt=\frac{\sqrt{2}}{2}\int_{0}^{1}\left(\frac{t}{t^2-\sqrt{2}t+1}-\frac{t}{t^2+\sqrt{2}t+1}\right)dt

Írhatjuk:

I=\frac{\sqrt{2}}{2}\int_{0}^{1}\left(\frac{t}{(t-\frac{\sqrt{2}}{2})^2+(\frac{\sqrt{2}}{2})^2}-\frac{t}{(t+\frac{\sqrt{2}}{2})^2+(\frac{\sqrt{2}}{2})^2}\right)dt

ahonnan kiszámítható I értéke:

I=\frac{\sqrt{2}}{4}\ln\frac{2-\sqrt{2}}{2+\sqrt{2}}+\frac{\sqrt{2}}{2}arctg{(\sqrt{2}+1)}+\frac{\sqrt{2}}{2}arctg{(\sqrt{2}-1)}

Előzmény: [1945] Lóczi Lajos, 2007-03-20 03:00:23
[1945] Lóczi Lajos2007-03-20 03:00:23

311. feladat. Mekkora a tangensfüggvény négyzetgyökének görbe alatti területe 0 és \pi/4 között?

[1944] Fálesz Mihály2007-03-13 13:43:26

A zeta-függvény szorzat alakjából és néhány ismert értékéből kijön.

\zeta(s)=\prod_p\frac1{1-\frac1{p^s}} ~~~~ ({\rm re}~s>1)

.

\prod_p\left(1-\frac1{p^2}\right)=\frac1{\zeta(2)}=\frac6{\pi^2},

\prod_p\left(1-\frac1{p^4}\right)=\frac1{\zeta(4)}=\frac{90}{\pi^4},


\prod_p\frac{p^2+1}{p^2-1}=
\prod_p\frac{1-\frac1{p^4}}{\big(1-\frac1{p^2}\big)^2}
=\frac{90/\pi^4}{(6/\pi^2)^2}=\frac52.

Előzmény: [1941] Lóczi Lajos, 2007-03-10 18:10:09
[1943] Csimby2007-03-11 13:39:10

309.feladat Van e olyan valós szám amelynek bármely egész számrendszerben felírt alakjában minden számjegy szerepel legalább egyszer.

310.feladat Igaz-e, hogy additív halmazfüggvények szorzata is additív?

Amúgy nincs kedvetek visszatérni a feladatok sorszámozásához? Szerintem az olyan jól bejött eddig...

[1942] ágica2007-03-10 21:13:18

Eszerint az oldal szerint a szorzat értéke 5/2, és az eredmény Ramanujantól származik:

http://mathworld.wolfram.com/PrimeProducts.html

Az első 30-40 prímre kiszámolva a szorzatot egyébként már elég jól meg is sejthető az eredmény.

Előzmény: [1941] Lóczi Lajos, 2007-03-10 18:10:09
[1941] Lóczi Lajos2007-03-10 18:10:09

Elképzelhető, hogy mi nem tudjuk megoldani a feladatot, mert nehéz; azért írtam be csak, mert meglepően egyszerű s szép a végeredménye. Inkább sejtsük meg numerikusan, vagy keressük meg az interneten az értékét, illetve, hogy ki az, aki rátalált erre a formulára. (Bár azt is el tudom képzelni, hogy igazából nem is számít, hogy pontosan prímekről van benne szó, és esetleg elég lenne egy alsó/felső becslés az n-edik prímszámra, és abból is ugyanaz az érték jönne ki?)

Előzmény: [1940] Cckek, 2007-03-10 17:25:37
[1940] Cckek2007-03-10 17:25:37

Ha L-el jelöljük a határértéket:

e^{\sum_{n\ge 1}\frac{2}{p_n^2+1}}\le L\le e^{\sum_{n\ge 1}\frac{2}{p_n^2-1}}, de innen tovább???

Előzmény: [1939] Lóczi Lajos, 2007-03-09 23:53:09
[1939] Lóczi Lajos2007-03-09 23:53:09

Jelölje pn az n-edik prímszámot.

Határozzuk meg a \prod_{n=1}^\infty \frac{p_n^2+1}{p_n^2-1} végtelen szorzat értékét.

[1938] Cckek2007-03-09 07:09:57

Felhasználva hogy

\sqrt{a}-\sqrt{b}=\frac{a-b}{\sqrt{a}+\sqrt{b}}

kapjuk

\frac{\sqrt{n+1}-\sqrt{n-1}}{\sqrt{n+1}-\sqrt{n}}=\frac{2(\sqrt{n+1}+\sqrt{n})}{\sqrt{n+1}+\sqrt{n-1}}=2\frac{\sqrt{1+\frac1n}+1}{\sqrt{1+\frac1n}+\sqrt{1-\frac1n}}

Előzmény: [1936] lorantfy, 2007-03-07 10:06:58
[1937] S.Ákos2007-03-07 20:07:40

Nem tudom, szerintem nem jó. Például n=1-re \frac12et ad, ami az intervallumok számának egész volta miatt lehetetlen.

Előzmény: [1932] Cckek, 2007-03-01 23:37:51
[1936] lorantfy2007-03-07 10:06:58

Bocs elírtam! (Zöld könyv 3676. példa)

Előzmény: [1935] Sirpi, 2007-03-07 08:01:09
[1935] Sirpi2007-03-07 08:01:09

Mármint -2 :-)

Előzmény: [1934] lorantfy, 2007-03-06 22:24:30
[1934] lorantfy2007-03-06 22:24:30

Egy feladat a Zöld könyvből:

[1933] Doom2007-03-06 07:29:59

Legutóbb rossz helyre tettem, remélem itt már jó...

Egy 'a' oldalú szabályos háromszög minden csúcsában 1-1 kutya áll (A, B és C), majd egyszerre elkezdenek futni egymás felé azonos sebességgel: A B felé, B C felé és C pedig A irányában. Mennyi idő múlva találkoznak?

Segítségként egy "sejtés": egy furcsa "spirál-alakot" megtéve a háromszög középpontjában fognak találkozni, mégpedig egyszerre.

  [1]    [2]    [3]    [4]    [5]    [6]    [7]    [8]    [9]    [10]    [11]    [12]    [13]    [14]    [15]    [16]    [17]    [18]    [19]    [20]    [21]    [22]    [23]    [24]    [25]    [26]    [27]    [28]    [29]    [30]    [31]    [32]    [33]    [34]    [35]    [36]    [37]    [38]    [39]    [40]    [41]    [42]    [43]    [44]    [45]    [46]    [47]    [48]    [49]    [50]    [51]    [52]    [53]    [54]    [55]    [56]    [57]    [58]    [59]    [60]    [61]    [62]    [63]    [64]    [65]    [66]    [67]    [68]    [69]    [70]    [71]    [72]    [73]    [74]    [75]    [76]    [77]    [78]    [79]    [80]    [81]    [82]    [83]    [84]    [85]    [86]    [87]    [88]    [89]    [90]    [91]    [92]    [93]    [94]    [95]    [96]    [97]    [98]    [99]    [100]    [101]    [102]    [103]    [104]    [105]    [106]    [107]    [108]    [109]    [110]    [111]    [112]    [113]    [114]    [115]    [116]    [117]    [118]    [119]    [120]    [121]    [122]    [123]    [124]    [125]    [126]    [127]    [128]    [129]    [130]    [131]    [132]    [133]    [134]    [135]    [136]    [137]    [138]    [139]    [140]    [141]    [142]    [143]    [144]    [145]    [146]    [147]    [148]    [149]    [150]    [151]    [152]    [153]    [154]    [155]    [156]    [157]    [158]    [159]    [160]    [161]